0 Daumen
871 Aufrufe

Sei \( n \in \mathbb{N}_{\geq 1} \). Zeigen Sie

\( \sum \limits_{k=0}^{n} \frac{(-1)^{k}}{k+1}\left(\begin{array}{l} n \\ k \end{array}\right)=\frac{1}{n+1} \)

Ansatz:
Dass man das mit der vollständigen Induktion beweist, ist klar. Induktionsanfang haut hin, aber beim Induktionsschritt stehe ich dann vor dem Problem.
Dass die Summe bis n+1 gehen muss, weiß ich. Auch im Binomialkoeffizient kommt n+1 hin. Aber wie formt man dann um? Wir haben zwar die Additionsregel für Binomialkoeffizienten gelernt, aber dann erhalte ich immer wieder eine Summe mit einem neuen Binomialkoeffizienten. Und das ist genau das Problem. Weiß jemand, wie man (anders) vorgehen kann?

Danke schon einmal

Avatar von

Es gilt vermutlich:

(n + 1 über k) = (n über k)·(n + 1)/(n - k + 1)

Ich habe aber nicht probiert, ob das zum Ziel führt. Aber so würde ich es erstmal probieren.

Musst oder möchtest Du mit VInd beweisen??

Kann. Es steht nicht genau da wie, es klingt doch aber nach vollst. Induktion, oder?

1 Antwort

0 Daumen
 
Beste Antwort

Ohne Induktion geht es so: Wir nehmen n+1 mit auf die linke Seite. Zunächst:

$$\frac{n+1}{k+1}{n \choose k}=\frac{n+1}{k+1}\frac{n!}{k!(n-k)!}=\frac{(n+1)!}{(k+1)!(n+1-(k+1))!}={n+1 \choose k+1}$$

Damit

$$\sum_{k=0}^n(-1)^k\frac{n+1}{k+1}{n \choose k}=-\sum_{k=0}^n(-1)^{k+1}{n+1 \choose k+1}$$

$$=1-\sum_{k=0}^{n +1}(-1)^{k}{n+1 \choose k}=1-(1-1)^{n+1}=1$$

Avatar von 13 k

Ein anderes Problem?

Stell deine Frage

Willkommen bei der Mathelounge! Stell deine Frage einfach und kostenlos

x
Made by a lovely community